2018年9月4日 星期二

一百零一學年度數學學科能力測驗

大學入學考試中心
$101$學年度學科能力測驗試題

數學考科



-作答注意事項-

  1. 考試時間:$100$分鐘
  2. 題型題數:單選題$7$題,多選題$6$題,選填題第$A$至$G$題共$7$題
  3. 作答方式:用2B鉛筆在「答案卡」上作答;更正時,應以橡皮擦擦拭,切勿使用修正液(帶)。未依規定畫記答案卡,致機器掃描無法辨識答案者,其後果由考生自行承擔。
  4. 選填題作答說明:選填題的題號是A,B,C,……,而答案的格式每題可能不同,考生必須依各題的格式填答,且每一個列號只能在一個格子畫記。請仔細閱讀下面的例子。

    例:若第B題的答案格式是$\displaystyle\underline{ \frac{⑱}{⑲} }$,而依題意計算出來的答案是$\displaystyle\frac{3}{8}$,則考生必須分別在答案卡上的第$18$列的$\underset{\boxed{~~}}{3}$與第$19$列的$\underset{\boxed{~~}}{8}$劃記,如:

    $\begin{array}{|c|}18~~\underset{\boxed{~~}}{-}~\underset{\boxed{~~}}{±}~\underset{\boxed{~~}}{1}~\underset{\boxed{~~}}{2}~\underset{\color{black}{▆▆}}{3}~\underset{\boxed{~~}}{4}~\underset{\boxed{~~}}{5}~\underset{\boxed{~~}}{6}~\underset{\boxed{~~}}{7}~\underset{\boxed{~~}}{8}~\underset{\boxed{~~}}{9}~\underset{\boxed{~~}}{0}\\19~~\underset{\boxed{~~}}{-}~\underset{\boxed{~~}}{±}~\underset{\boxed{~~}}{1}~\underset{\boxed{~~}}{2}~\underset{\boxed{~~}}{3}~\underset{\boxed{~~}}{4}~\underset{\boxed{~~}}{5}~\underset{\boxed{~~}}{6}~\underset{\boxed{~~}}{7}~\underset{\color{black}{▆▆}}{8}~\underset{\boxed{~~}}{9}~\underset{\boxed{~~}}{0}\end{array}$

    例:若第C題的答案格式是$\displaystyle\underline{ \frac{⑳㉑}{50} }$,而答案是$\displaystyle\frac{-7}{50}$時,則考生必須分別在答案卡的第$20$列的$\underset{\boxed{~~}}{-}$與第$21$列的$\underset{\boxed{~~}}{7}$畫記,如:

    $\begin{array}{|c|}20~~\underset{\color{black}{▆▆}}{-}~\underset{\boxed{~~}}{±}~\underset{\boxed{~~}}{1}~\underset{\boxed{~~}}{2}~\underset{\boxed{~~}}{3}~\underset{\boxed{~~}}{4}~\underset{\boxed{~~}}{5}~\underset{\boxed{~~}}{6}~\underset{\boxed{~~}}{7}~\underset{\boxed{~~}}{8}~\underset{\boxed{~~}}{9}~\underset{\boxed{~~}}{0}\\\hline21~~\underset{\boxed{~~}}{-}~\underset{\boxed{~~}}{±}~\underset{\boxed{~~}}{1}~\underset{\boxed{~~}}{2}~\underset{\boxed{~~}}{3}~\underset{\boxed{~~}}{4}~\underset{\boxed{~~}}{5}~\underset{\boxed{~~}}{6}~\underset{\color{black}{▆▆}}{7}~\underset{\boxed{~~}}{8}~\underset{\boxed{~~}}{9}~\underset{\boxed{~~}}{0}\end{array}$

  5. ※試題後附有參考公式及可能用到的數值


第壹部分:選擇題(占$65$分)
  1. 單選題(占$35$分)
  2. 說明:第$1$題至第$7$題,每題有$5$個選項,其中只有一個是正確或最適當的選項,請畫記在答案卡之「選擇(填)題答案區」。各題答對者,得$5$分;答錯、未作答或畫記多於一個選項者,該題以零分計算。
    1. $\displaystyle\sqrt{\frac{1}{5^2}+\frac{1}{4^2}+1}$等於下列哪一個選項?
      1. $1.01$
      2. $1.05$
      3. $1.1$
      4. $1.15$
      5. $1.21$
    2. 訣竅直接計算即可。
      解法直接計算有

      $\displaystyle\sqrt{\frac{1}{5^2}+\frac{1}{4^2}+1}=\sqrt{\frac{1}{25}+\frac{1}{16}+1}=\sqrt{\frac{16+25+400}{400}}=\sqrt{\frac{441}{400}}=\frac{21}{20}=1.05$

      故選(2)。

    3. 將邊長為$1$公分的正立方體堆疊成一階梯形立體,如下圖所示,其中第$1$層(最下層)有$10$塊,第$2$層有$9$塊,…,依此類推。當堆疊完$10$層時,該階梯形立體的表面積(即該立體的前、後、上、下、左、右各表面的面積總和)為多少?
      1. $75$平方公分
      2. $90$平方公分
      3. $110$平方公分
      4. $130$平方公分
      5. $150$平方公分
    4. 訣竅在計算表面積時要注意到僅需計算正前方、左側方和下側方三者之總和後的兩倍。
      解法注意到前面的表面積與後面的表面積相同,而左面的表面積與右面的表面積也相同,上面的表面積也與下面的表面積相同。因此總表面積為

      $\left(55+10+10\right)\times2=150$平方公分

      應選(5)。

    5. 下表為常用對數表$\log_{10}N$的一部分:\begin{array}{|c|ccccc|ccccc|}\hline N&0&1&2&3&4&5&6&7&8&9\\10&0000&0043&0086&0128&0170&0212&0253&0294&0334&0374\\11&0414&0453&0492&0531&0569&0607&0645&0682&0719&0755\\\vdots&\vdots&\vdots&\vdots&\vdots&\vdots&\vdots&\vdots&\vdots&\vdots&\vdots\\20&3010&3032&3054&3075&3096&3118&3139&3160&3181&3201\\\vdots&\vdots&\vdots&\vdots&\vdots&\vdots&\vdots&\vdots&\vdots&\vdots&\vdots\\30&4771&4786&4800&4814&4829&4843&4857&4871&4886&4900\\\hline\end{array}請問$10^{3.032}$最接近下列哪一個選項?
      1. $101$
      2. $201$
      3. $1007$
      4. $1076$
      5. $2012$
    6. 訣竅運用指數律確定應分析的對象後利用對數表估算其範圍。
      解法根據對數表可知$\log1.07\approx0.0294$、$\log_1.08\approx0.0334$,此表明$1.07<10^{0.032}<1.08$,故依$10^{3.032}=10^3\cdot10^{0.032}$可知

      $1070<10^{3.032}<1080$

      故選(4)。

    7. 甲、乙兩校有一樣多的學生參加數學能力測驗,兩校學生測驗成績的分布都很接近常態分布,其中甲校學生的平均分數為$60$分,標準差為$10$分;乙校學生的平均分數為$65$分,標準差為$5$分。若用粗線表示甲校學生成績分布曲線;細線表示乙校學生成績分布曲線,則下列哪一個分布圖較為正確?
    8. 訣竅瞭解平均分數與標準差對圖形的影響。
      解法由於甲校的平均分數較乙校低,因此其高峰的位置較乙校偏左;又甲校的標準差較乙校大,因此其圖形較為矮寬,故選(1)。

    9. 若正實數$x,y$滿足$\log_{10}x=2.8$,$\log_{10}y=5.6$,則$\log_{10}\left(x^2+y\right)$最接近下列哪一個選項的值?
      1. $2.8$
      2. $5.6$
      3. $5.9$
      4. $8.4$
      5. $11.2$
    10. 訣竅運用對數的定義求解。
      解法按定義可知$x=10^{2.8}$、$y=10^{5.6}$,故$x^2+y=2\times10^{5.6}$,因此

      $\log{10}\left(x^2+y\right)=\log_{10}2+\log_{10}10^{5.6}\approx5.6+0.301=5.901$

      應選(3)。

    11. 箱中有編號分別為$0,1,2,\cdots,9$的十顆球。隨機抽取一球,將球放回後,再隨機抽取一球。請問這兩球編號相減的絕對值為下列哪一個選項時,其出現的機率最大?
      1. $0$
      2. $1$
      3. $4$
      4. $5$
      5. $9$
    12. 訣竅釐清各選項之方法數,選出有最多方法數者即可。
      解法前後兩次取球的情形共有$100$種狀況,兩球編號相減的絕對值為各個選項之可能數如下
      • 相差為$0$者有$10$種,即$\left(0,0\right),\cdots,\left(9,9\right)$;
      • 相差為$1$者有$18$種,即$\left(0,1\right),\left(1,2\right),\cdots,\left(8,9\right)$與$\left(1,0\right),\left(2,1\right),\cdots,\left(9,8\right)$;
      • 相差為$4$者有$12$種,即$\left(0,4\right),\left(1,5\right),\cdots,\left(5,9\right)$與$\left(4,0\right),\left(5,1\right),\cdots,\left(9,5\right)$
      • 相差為$5$者有$10$種,即$\left(0,5\right),\left(1,6\right),\cdots,\left(4,9\right)$與$\left(5,0\right),\left(6,1\right),\cdots,\left(9,4\right)$。
      • 相差為$9$者共$2$種,即$\left(0,9\right)$與$\left(9,0\right)$。
      故選(2)。

    13. 空間坐標中有一球面(半徑大於$0$)與平面$3x+4y=0$相切於原點,請問此球面與三個坐標軸一共有多少個交點?
      1. $1$
      2. $2$
      3. $3$
      4. $4$
      5. $5$
    14. 訣竅按條件思索球心的位置,如此作圖後容易觀察球心分別與各軸相交的情形。
      解法由於球面與平面$3x+4y=0$切於原點,可以知道球心落在平面$z=0$上,從而球面與$x$軸交兩點、$y$軸交兩點與$z$軸恰交一點,其中三者共同為原點,故共有$3$點,應選(3)。
  3. 多選題(占$30$分)
  4. 說明:第$8$題至第$13$題,每題有$5$個選項,其中至少有一個是正確的選項,請將正確選項畫記在答案卡之「選擇(填)題答案區」。各題之選項獨立判定,所有選項均答對者,得$5$分;答錯$1$個選項者,得$3$分;答錯$2$個選項者,得$1$分;答錯多於$2$個選項或所有選項均未作答者,該題以零分計算。
    1. 設$f\left(x\right)=x^4-5x^3+x^2+ax+b$為實係數多項式,且知$f\left(i\right)=0$(其中$i^2=-1$)。請問下列哪些選項是多項式方程式$f\left(x\right)=0$的根?
      1. $-i$
      2. $0$
      3. $1$
      4. $-5$
      5. $5$
    2. 訣竅利用實係數多項式方程式虛根成對以及因式定理確定$f\left(x\right)$,隨後透過因式分解求出所有根。
      解法由虛根成對定理可知$-i$亦為根,從而有因式$\left(x-i\right)$與$\left(x+i\right)$,故$f\left(x\right)$有因式$x^2+1$,因此作長除法有

      $x^4-5x^3+x^2+ax+b=\left(x^2+1\right)\left(x^2-5x\right)+\left(a-5\right)x+b$

      由此可知$a=5$、$b=0$,即有

      $f\left(x\right)=x\left(x-5\right)\left(x^2+1\right)$

      故四根分別為$0,5,i,-i$,應選(1)(2)(5)。

    3. 三角形$ABC$是一個邊長為$3$的正三角形,如下圖所示。若在每一邊的兩個三等分點中,各選取一點連成三角形,則下列哪些選項是正確的?
      1. 依此方法可能連成的三角形一共有$8$個
      2. 這些可能連成的三角形中,恰有$2$個是銳角三角形
      3. 這些可能連成的三角形中,恰有$3$個是直角三角形
      4. 這些可能連成的三角形中,恰有$3$個是鈍角三角形
      5. 這些可能連成的三角形中,恰有$1$個是正三角形
    4. 訣竅直接繪圖清點即可。
      解法為方便起見,在$\overline{AB}$、$\overline{BC}$、$\overline{CA}$邊上分別依逆時針記這六個點分別為$D$、$E$、$F$、$G$、$H$、$I$。據此可以發現如下的事實:
      • 由於各邊各取一點,因此各有兩種選擇,從而有$2\times2\times2=8$種,亦可列舉如下:

        $\Delta DFH,~\Delta DFI,~\Delta DGH,~\Delta DGI,~\Delta EFH,~\Delta EFI,~\Delta EGH,~\Delta EGI$

      • 由前項事實中可以發現$\Delta DFH,~\Delta EGI$兩者皆為正三角形,即有$2$個銳角三角形、$2$個正三角形。
      • $\Delta DFI,~\Delta DGH,~\Delta DGI,~\Delta EFH,~\Delta EFI,~\Delta EGH$等六個三角形為直角三角形。
      根據以上的結果可知應選(1)(2)。

    5. 設$O$為複數平面上的原點,並令點$A$,$B$分別代表非零複數$z$,$w$。若$\angle AOB=90^\circ$,則下列哪些選項必為負實數
      1. $\displaystyle\frac{z}{w}$
      2. $zw$
      3. $\left(zw\right)^2$
      4. $\displaystyle\frac{z^2}{w^2}$
      5. $\left(z\bar{w}\right)^2$(其中$\bar{w}$為$w$的共軛複數)
    6. 訣竅根據條件進行假設後根據複數的乘除計算檢查之。
      解法按條件可設$z=r_1\left(\cos\theta+i\sin\theta\right)$,$w=r_2\left[\cos\left(\theta\pm90^\circ\right)+i\sin\left(\theta\pm90^\circ\right)\right]$,那麼可以計算各個選項如下:
      1. $\displaystyle\frac{z}{w}=\frac{r_1}{r_2}\left(\cos\pm90^\circ+i\sin\pm90^\circ\right)=\pm\frac{r_1}{r_2}i$為純虛數。
      2. $zw=r_1r_2\left[\cos\left(2\theta\pm90^\circ\right)+i\sin\left(2\theta\pm90^\circ\right)\right]$,不必然為負實數。
      3. $\left(zw\right)^2=r_1^2r_2^2\left[\cos\left(4\theta\pm180^\circ\right)+i\sin\left(4\theta\pm180^\circ\right)\right]$,不必然是負實數。
      4. $\displaystyle\frac{z^2}{w^2}=\left(\frac{z}{w}\right)^2=-\frac{r_1^2}{r_2^2}<0$為負實數。
      5. $\left(z\bar{w}\right)^2=\left[r_1r_2\cos\left(\mp90^\circ\right)+i\sin\left(\mp90^\circ\right)\right]^2=-r_1^2r_2^2<0$為負實數。
      故應選(4)(5)。

    7. 若實數$a,b,c,d$使得聯立方程組$\left\{\begin{aligned} &ax+8y=c\\&x-4y=3\end{aligned}\right.$有解,且聯立方程組$\left\{\begin{aligned} &-3x+by=d\\&x-4y=3\end{aligned}\right.$無解,則下列哪些選項一定正確?
      1. $a\neq-2$
      2. $c=-6$
      3. $b=12$
      4. $d\neq-9$
      5. 聯立方程組$\left\{\begin{aligned} &ax+8y=c\\&-3x+by=d\end{aligned}\right.$無解
    8. 訣竅注意聯立方程組的無解的條件,並且有解可能是恰有一解或是無窮多解。此外也應注意聯立方程組所表示的幾何意義。
      解法

      首先處理聯立方程組$\left\{\begin{aligned} &ax+8y=c\\&x-4y=3\end{aligned}\right.$,使用加減消去法有$\left(a+2\right)x=c+6$。由於方程組有解可能恰有一解,則此時有$a\neq-2$;而當方程組的有解為無窮多解時,則有$a=-2$、$c=-6$。

      另一方面,聯立方程組$\left\{\begin{aligned} &-3x+by=d\\&x-4y=3\end{aligned}\right.$是無解的,那麼由加減消去法可得的方程式也無解:

      $\left(b-12\right)=d+9$

      因此$b=12$但$d\neq-9$。

      最後由直線們的幾何意義可以分出兩種情形:$ax+8y=c$與$x-4y=3$重合還是恰交一點,兩種情形分別可知聯立方程組$\left\{\begin{aligned} &ax+8y=c\\&-3x+by=d\end{aligned}\right.$無解或恰有一解。

      至此可以知道一定正確的選項僅有(3)(4)。

    9. 在坐標平面上,廣義角$\theta$的頂點為原點$O$,始邊為$x$軸正向,且滿足$\displaystyle\tan\theta=\frac{2}{3}$。若$\theta$的終邊上有一點$P$,其$y$坐標為$-4$,則下列哪些選項一定正確?
      1. $P$的$x$坐標是$6$
      2. $\overline{OP}=2\sqrt{13}$
      3. $\displaystyle\cos\theta=\frac{3}{\sqrt{13}}$
      4. $\sin2\theta>0$
      5. $\displaystyle\cos\frac{\theta}{2}<0$
    10. 訣竅根據三角函數的定義以及倍角與半角公式等計算之。
      解法
      1. 設$P$之坐標為$\left(a,-4\right)$,那麼按廣義角三角函數的定義可知$\displaystyle\tan\theta=\frac{-4}{a}=\frac{2}{3}$,從而有$a=-6$,因此本選項錯誤。
      2. 由於$P$坐標為$\left(-6,-4\right)$,故$\overline{OP}=\sqrt{\left(-6\right)^2+\left(-4\right)^2}=\sqrt{52}=2\sqrt{13}$,本選項正確。
      3. 按廣義角三角函數的定義可知$\displaystyle\cos\theta=\frac{x}{r}=\frac{-6}{2\sqrt{13}}=\frac{-3}{\sqrt{13}}$,本選項錯誤。
      4. 由二倍角公式可知

        $\displaystyle\sin2\theta=2\sin\theta\cos\theta=2\cdot\frac{-2}{\sqrt{13}}\cdot\frac{-3}{\sqrt{13}}=\frac{12}{13}>0$

        本選項正確。
      5. 注意到$\sin\theta$、$\cos\theta$皆為負值,因此$\theta$為第三象限角,即$\theta$介於$360^\circ\cdot k+180^\circ$與$360^\circ\cdot k+270^\circ$之間,其中$k$為整數,從而$\displaystyle\frac{\theta}{2}$介於$180^\circ\cdot k+90^\circ$與$180^\circ\cdot k+135^\circ$之間。當$k=1$時表明$\displaystyle\frac{\theta}{2}$為第四象限角,從而$\displaystyle\cos\frac{\theta}{2}>0$,因此本選項不一定正確。
      由以上可知應選(2)(4)。

    11. 平面上兩點$F_1,F_2$滿足$\overline{F_1F_2}=4$。設$d$為一實數,令$\Gamma$表示平面上滿足$\left|\overline{PF_1}-\overline{PF_2}\right|=d$的所有$P$點所成的圖形,又令$C$為平面上以$F_1$為圓心、$6$為半徑的圓。請問下列哪些選項是正確的?
      1. 當$d=0$時,$\Gamma$為直線
      2. 當$d=1$時,$\Gamma$為雙曲線
      3. 當$d=2$時,$\Gamma$與圓$C$交於兩點
      4. 當$d=4$時,$\Gamma$與圓$C$交於四點
      5. 當$d=8$時,$\Gamma$不存在。
    12. 訣竅注意到題目條件與雙曲線的定義式類似,並留意其極端情形。
      解法
      1. 當$d=0$時可以發現$P$落於$F_1$與$F_2$的中垂線上,因此$\Gamma$為直線,本選項正確。
      2. 而當$d=1<4$時可知$\Gamma$的圖形形成雙曲線,本選項正確。
      3. 當$d=2<4$時可知$\Gamma$的圖形形成雙曲線且方程為$\displaystyle\frac{x^2}{1}-\frac{y^2}{3}=1$,故交$x$軸於$\left(\pm1,0\right)$,藉由繪圖可發現雙曲線與$C$在四個象限中皆分別有一個交點,從而有四個交點,本選項錯誤。
      4. 當$d=4$時可知$\Gamma$為兩條分別以$F_1$、$F_2$為起點向外作的射線,從而與圓$C$分別交於$\left(-6,0\right)$與$\left(6,0\right)$,即有兩個交點,本選項錯誤。
      5. 而當$d=8>4$,從而$\Gamma$為空集合,因此無圖形或稱其不存在。本選項正確。
      由以上可知應選(1)(2)(5)。
第貳部分:選填題(占$35$分)
說明:
  1. 第$A$至$G$題,將答案畫記在答案卡之「選擇(填)題答案區」所標示的列號(14-33)處。
  2. 每題完全答對給$5$分,答錯不倒扣,未完全答對不給分。
  1. 若首項為$a$,公比為$0.01$的無窮等比級數和等於循環小數$1.\overline{2}$,則$a=\underline{ ⑭.⑮⑯ }$。
  2. 訣竅運用無窮等比級數公式並將循環小數化為有理數即可。
    解法首先注意到循環小數$1.\overline{2}$可改寫為$\displaystyle1+0.\overline{2}=1+\frac{2}{9}=\frac{11}{9}$,那麼題設與無窮等比級數公式可知

    $\displaystyle\frac{a}{1-0.01}=\frac{a}{0.99}=\frac{11}{9}$

    因此可知$\displaystyle a=0.99\times\frac{11}{9}=0.11\times11=1.21$,故填入$⑭=1$、$⑮=2$、$⑯=1$。

  3. 設$A\left(1,1\right)$,$B\left(3,5\right)$,$C\left(5,3\right)$,$D\left(0,-7\right)$,$E\left(2,-3\right)$及$F\left(8,-6\right)$為坐標平面上的六個點。若直線$L$分別與三角形$ABC$及三角形$DEF$各恰有一個交點,則$L$的斜率之最小可能值為 ⑰⑱
  4. 訣竅繪圖後觀察符合條件且使斜率為負又盡量傾斜的情形。
    解法
    如上附圖,為了使$L$斜率為負且盡量傾斜又僅與三角形各交一點,可推知應過$C$與$F$,從而斜率為$\displaystyle\frac{-6-3}{8-5}=\frac{-9}{3}=-3$,填入$⑰=-$、$⑱=3$。

  5. 小明在天文網站上看到以下的資訊「可利用北斗七星斗杓的天璇天樞這兩顆星來尋找北極星:由天璇起始向天樞的方向延伸便可找到北極星,其中天樞北極星的距離為天樞天璇距離的$5$倍。」今小明將所見的星空想像成一個坐標平面,其中天璇的坐標為$\left(9,8\right)$及天樞的坐標為$\left(7,11\right)$。依上述資訊可以推得北極星的坐標為$\left(\underline{ ⑲⑳ },\underline{ ㉑㉒ }\right)$。
  6. 訣竅將題意轉為向量的語言並注意該三星為共線。
    解法設天璇為$A$、天樞為$B$,北極星為$C$,那麼題意表明$\overset{\rightharpoonup}{BC}=5\overset{\rightharpoonup}{AB}$,因此有

    $C=B+5\left(B-A\right)=6B-5A=6\left(7,11\right)-5\left(9,8\right)=\left(-3,26\right)$

    因此填入$⑲=-$、$⑳=3$、$㉑=2$、$㉒=6$。

  7. 設點$A\left(-2,2\right)$、$B\left(4,8\right)$為坐標平面上兩點,且點$C$在二次函數$\displaystyle y=\frac{1}{2}x^2$的圖形上變動。當$C$點的$x$坐標為 ㉓㉔ 時,內積$\overset{\rightharpoonup}{AB}\cdot\overset{\rightharpoonup}{AC}$有最小值 ㉕㉖
  8. 訣竅藉由適當的假設$C$點座標並據此計算內積,從而利用配方法找出最小值,並根據等號成立條件找出對應的$C$點座標。
    解法設$C$之坐標為$\left(2a,2a^2\right)$,其中$a$為實數。那麼內積可以計算得

    $\displaystyle\overset{\rightharpoonup}{AB}\cdot\overset{\rightharpoonup}{AC}=\left(6,6\right)\cdot\left(2a+2,2a^2-2\right)=12a^2+12a=12\left(a+\frac{1}{2}\right)^2-3$

    如此可知當$\displaystyle a=-\frac{1}{2}$有最小值為$-3$,此時$C$之$x$坐標為$-1$,因此填入$㉓=-$、$㉔=1$、$㉕=-$、$㉖=3$。

  9. 在邊長為$13$的正三角形$ABC$上各邊分別取一點$P$,$Q$,$R$,使得$APQR$形成一平行四邊形,如下圖所示:若平行四邊形$APQR$的面積為$20\sqrt{3}$,則線段$PR$的長度為
  10. 訣竅為了使用餘弦定理,需弄清楚$\overline{AP}$、$\overline{AR}$之間的關係,如$\overline{AP}+\overline{AQ}$與$\overline{AP}\cdot\overline{AR}$。
    解法由於$APQR$為平行四邊形,因此其面積可表達如下

    $\overline{AP}\cdot\overline{AR}\cdot\sin60^\circ=20\sqrt{3}$

    即有$\overline{AP}\cdot\overline{AR}=40$。再者又由$\Delta BPQ$與$\Delta CQR$為正三角形,因此$AP+AR=QR+QP=CQ+QB=BC=13$。因此在$\Delta APR$中使用由餘弦定理有

    $\displaystyle\overline{PR}=\sqrt{\overline{AP}^2+\overline{AR}^2-2\overline{AP}\cdot\overline{AR}\cos60^\circ}=\sqrt{\left(13^2-2\cdot40\right)-2\cdot40\cdot\frac{1}{2}}=7$

    故填入$㉗=7$。

  11. 設$m,n$為正實數,橢圓$\displaystyle\frac{x^2}{m}+\frac{y^2}{n}=1$的焦點分別為$F_1\left(0,2\right)$與$F_2\left(0,-2\right)$。若此橢圓上有一點$P$使得$\Delta PF_1F_2$為一正三角形,則$m=\underline{ ㉘㉙ }$,$n=\underline{ ㉚㉛ }$。
  12. 訣竅留意橢圓標準式中各項參數的意義與相關的等式。
    解法按橢圓標準式的特性可知$n=m+2^2=m+4$。若橢圓上有點使$\Delta PF_1F_2$為正三角形,那麼由於$P$將落於$F_1F_2$的中垂線上,此表明$P$在$x$軸上。又$\overline{F_1F_2}=4$,故$\overline{PF_1}=\overline{PF_2}=4$,可以解得$P\left(\pm2\sqrt{3},0\right)$。將此坐標代入橢圓方程式中有$\displaystyle\frac{12}{m}=1$,即$m=12$,從而$n=16$,填入$㉘=1$、$㉙=2$、$㉚=1$、$㉛=6$。

  13. 坐標空間中,在六個平面$\displaystyle x=\frac{14}{13}$,$\displaystyle x=\frac{1}{13}$,$y=1$,$y=-1$,$z=-1$及$z=-4$所圍成的長方體上隨機選取兩個相異頂點。若每個頂點被選取的機率相同,則選到兩個頂點的距離大於$3$之機率為$\displaystyle\underline{ \frac{㉜}{㉝} }$。(化成最簡分數)
  14. 訣竅經由條件確定出長方體的長寬高後,探究各種選點所得到的距離的情形即可。
    解法經由條件可以知道長寬高分別為$3$、$2$、$1$,那麼任取一點時,至其餘七點的距離分別為$1$、$2$、$\sqrt{5}$、$3$、$\sqrt{10}$、$\sqrt{13}$、$\sqrt{14}$,故距離大於$3$的機率為$\displaystyle\frac{3}{7}$,應填入$㉜=3$、$㉝=7$

參考公式及可能用到的數值

  1. 一元二次方程式$ax^2+bx+c=0$的公式解:$\displaystyle x=\frac{-b\pm\sqrt{b^2-4ac}}{2a}$
  2. 平面上兩點$P_1\left(x_1,y_1\right)$,$P_2\left(x_2,y_2\right)$間的距離為$\overline{P_1P_2}=\sqrt{\left(x_2-x_1\right)^2+\left(y_2-y_1\right)^2}$
  3. 通過$\left(x_1,y_1\right)$與$\left(x_2,y_2\right)$的直線斜率$\displaystyle m=\frac{y_2-y_1}{x_2-x_1}$,$x_2\neq x_1$
  4. 首項為$a_1$,公差為$d$的等差數列前$n$項之和為$\displaystyle S=\frac{n\left(a_1+a_n\right)}{2}=\frac{n\left(2a_1+\left(n-1\right)d\right)}{2}$
    等比數列$\left\langle ar^{k-1}\right\rangle$的前$n$項之和$\displaystyle S_n=\frac{a\left(1-r^n\right)}{1-r}$,$r\neq1$
  5. 級數公式:$\displaystyle\sum_{k=1}^{n}k^2=1^2+2^2+3^2+\cdots+n^2=\frac{n\left(n+1\right)\left(2n+1\right)}{6}$
  6. 三角函數的和角公式:$\sin\left(A+B\right)=\sin A\cos B+\sin B\cos A$
              $\cos\left(A+B\right)=\cos A\cos B-\sin A\sin B$
  7. $\Delta ABC$的正弦定理: $\displaystyle\frac{a}{\sin A}=\frac{b}{\sin B}=\frac{c}{\sin C}=2R$,$R$為$\Delta ABC$的外接圓半徑
    $\Delta ABC$的餘弦定理: $c^2=a^2+b^2-2ab\cos C$
  8. 棣美弗定理:設$z=r\left(\cos\theta+i\sin\theta\right)$,則$z^n=r^n\left(\cos n\theta+i\sin n\theta\right)$,$n$為一正整數
  9. 算術平均數: $\displaystyle M\left(={\bar X}\right)=\frac{1}{n}\left(x_1+x_2+\cdots+x_n\right)=\frac{1}{n}\sum_{i=1}^{n}x_i$
    (樣本)標準差: $\displaystyle S=\sqrt{\frac{1}{n-1}\sum_{i=1}^{n}\left(x_i-{\bar X}\right)^2}=\sqrt{\frac{1}{n-1}\left(\left(\sum_{i=1}^{n}x_i^2\right)-n\bar{X}^2\right)}$
  10. $95\%$信心水準下的信賴區間:$\displaystyle\left[\hat{p}-2\sqrt{\frac{\hat{p}\left(1-\hat{p}\right)}{n}},\hat{p}+2\sqrt{\frac{\hat{p}\left(1-\hat{p}\right)}{n}}\right]$
  11. 參考數值:$\sqrt{2}\approx1.414$; $\sqrt{3}\approx1.732$; $\sqrt{5}\approx2.236$; $\sqrt{6}\approx2.449$; $\pi\approx3.142$
  12. 對數值:$\log_{10}2\approx0.3010$,$\log_{10}3\approx0.4771$,$\log_{10}5\approx0.6990$,$\log_{10}7\approx0.8451$

沒有留言:

張貼留言